3x-5y=11 if x equals -1,0,1

Answers

Answer 1

Answer:

 x = 5.33333333333

Step-by-step explanation:

3x-5=11

   +5   +5

3x= 16

3      3      

u need to separate the 3x

so u use the opposite operation on - 5

add +5 on both sides

-5 plus 5 cancel each other

add 11 to 5 and that is 16

bring them down and divide both by 3

and ur 3 cancel each other,

now divide 16 by 3

and u get  x = 5.33333333333


Related Questions

A cash discount of 8% is allowed on an item which costs £45. How much money is saved if a customer decides to pay in cash? How much more can he save if the discount is 9%?

Answers

well, if she decides to pay in cash, the savings is 8%!!!! yeah!!!, well hell how much is that?

[tex]\begin{array}{|c|ll} \cline{1-1} \textit{\textit{\LARGE a}\% of \textit{\LARGE b}}\\ \cline{1-1} \\ \left( \cfrac{\textit{\LARGE a}}{100} \right)\cdot \textit{\LARGE b} \\\\ \cline{1-1} \end{array}~\hspace{5em}\stackrel{\textit{8\% of 45}}{\left( \cfrac{8}{100} \right)45}\implies \text{\LARGE \pounds 3.6}[/tex]

what if 9%? well, how much is 9%?

[tex]\begin{array}{|c|ll} \cline{1-1} \textit{\textit{\LARGE a}\% of \textit{\LARGE b}}\\ \cline{1-1} \\ \left( \cfrac{\textit{\LARGE a}}{100} \right)\cdot \textit{\LARGE b} \\\\ \cline{1-1} \end{array}~\hspace{5em}\stackrel{\textit{9\% of 45}}{\left( \cfrac{9}{100} \right)45}\implies 4.05~\hfill \underset{\textit{more in savings}}{\stackrel{4.05~~ - ~~3.6}{\text{\LARGE \pounds 0.45}}}[/tex]

4x12=6 x n guys please help me on this im in 4th grade and dont know it

Answers

Answer:

4x12=6x

48=6x

x=48/6

x=8

4 • 12 = 6x

•first we have like terms on the same side of the equal sign so this allows us to combine them

48 = 6x

• next we must get the x alone so we will divide the x to both sides. (Remember if it’s addition you subtract and if it’s multiplication you divide and vice versa)

In this case we divide because it’s multiplication:

48 = 6x
__ __
6 6

Cross the 6 out so we have x alone

This gives us 8 = x




ABCDE is a rectangular-based pyramid with a height of 14 cm.
13 cm
E
Submit Answer
1
0
D
Find the angle between the line DE and the plane ABCD.
Give your answer correct to 1 decimal place.
C
10 cm.

Answers

Answer:

59.6°

Step-by-step explanation:

Write the second ligament definition above as a biconditional. Use "if" and "only if".

2. A ligament is a bond of tissue connecting bones or holding organs in place.

Answers

Using  "if" and "only if" for the second definition of ligament gives:

A ligament is a bond of tissue if and only if, it connects bones or hold organs in place.

What is a biconditional statement?

This is a term used to refer to a type o statement that brings two conditional statement together, Most of the times biconditional statements use if and only if to combine the two conditions.

In the given statement, the two conditions are

A ligament is a bond of tissue ligaments connects bones or hold organs in place.

Biconditional statements ensures that the two statements are maintained for the statement to be valid.

Read more on Biconditional statements here: https://brainly.com/question/24528571

#SPJ1

A triangle has vertices at A (-2, -2), B (-1, 1), and C (3, 2). Which of the following transformations produces an image with vertices A' (2, -2), B' (1, 1), and C' (-3, 2)?
O (x, y) -> (x, - y)
O (x, y) -› (-y, x)
O(x,y) → (-x,y)
O (x, y) -> (y, -x)

Answers

Answer:

[tex](x, y) \rightarrow (-x, y)[/tex]

(Option 3)

Step-by-step explanation:

Let's take into account ΔABC.

A(-2, -2) B(-1, 1) C(3, 2)

This process is entirely trial and error. Try out the different transformations and see which one results the image from the preimage.

In this case, our transformation is (-x, y). Let's put this into trial and error:

[tex]A(-2, -2) \rightarrow A'(2, -2)\\B(-1, 1) \rightarrow B'(1, 1)\\C(3, 2) \rightarrow C'(-3, 2)[/tex]

Simply, the values we got for the image is the same as provided in the question. So we know that option 3 is the right answer.

Answer:

Mackenzie the answer is the 3rd option:)

Step-by-step explanation:

Find the y-intercept of the line 15x+y=-4
Write your answer as an integer or as a simplified proper or improper fraction, not as an ordered pair.

Answers

u vu h,j h j h jcuvugogjjvjccjcjcvk

FISRT RIGHT THEN EQUATION IN THE GENERAL FORM y=mx+c

y=-4-15x

In the y intercept the x coordinate in the axis is 0.

therefore we can plug in the value 0 in the place of x and solve for y

[tex]y = - 4 - 15(0) \\ y = - 4[/tex]

The y intercept is -4.

if your heart beats 700 times in 10 minutes how many times in 2 minutes

Answers

Answer:

your heart beats 140 times in two mintutes  or 70 times per minute

Step-by-step explanation:

to find how many beats per minute :

700/10= 70

and then to find for two minutes :

70x2= 140

Hope this helped

21. If x: y = 3: 4, find (3x + 4y): (5x + 6y).


please fast answer I will mark you brain list answer ​

Answers

Answer: Correct option is B)

Put x=3y=4

(3x+4y):(5x+6y)=(3×3+4×4):(5×3+6×4)

                                      =(9+16):(15+24)

                                      =25:39

Step-by-step explanation:

evaluate the product $$ (\sqrt{5} \sqrt{6} \sqrt{7}) (\sqrt{5} \sqrt{6} - \sqrt{7}) (\sqrt{5} - \sqrt{6} \sqrt{7}) (-\sqrt{5} \sqrt{6} \sqrt{7}) $$

Answers

By applying algebra, radical and power properties, we find that the simplified form of (√5 + √6) · (√7 - √5) is equal to √35 + √42 - 5 - √30.

How to simplify a product of two binomials with radical numbers

Herein we need to apply algebra and radical and power properties to simplify the expression described in the statement, whose procedure is shown below:

(√5 + √6) · (√7 - √5)     Given

(√5 + √6) · √7 + (√5 + √6) · (- √5)     Distributive property

√7 · √5 + √6 · √7 + √5 · (- √5) + √6 · (- √5)     Distributive property

√35 + √42 - 5 - √30     (- a) · b = - a · b/ √a · √b = √a · b/Result

By applying algebra, radical and power properties, we find that the simplified form of (√5 + √6) · (√7 - √5) is equal to √35 + √42 - 5 - √30.

To learn more about of  radical numbers here

https://brainly.com/question/27980015

#SPJ4

Got the answer to a already just need b

Answers

a) Number of points the team has  is 18

b) Number of games lost by the team is  9

a) How many point does the team have?

Number of wins the team has  = W= 5

Number of draws the team has  = D= 3

Number of points of the team = P

The equation is given by,

P = 3W +D

P= 3*5 + 3

P = 15+3

P = 18

Number of points the team has 18 points

b) How many games has this team lost?

Number of points of the team =P = 50

Number of draws the team has = D= 14

Number of wins the team has  = W

Number of matches played by the team =T= 35

The equation is given by,

P = 3W +D

50 = 3W+ 14

3W = 50- 14

3W = 36

W = 12

Rewriting as equation,

Lost matches = Total matches - (Win matches+ Draw matches)

                      = T - (W+ D)

                      = 35 - (12+ 14)

                      =35 - 26

                      = 9

Number of games lost by the team = 9

What is an equation?

An equation declares that two things are equal. It will include an equals sign ' ='  similar to this. The terms left-hand side and right-hand side refer to the expressions on each side of the equals sign. It's very common to presume that an equation's right side is zero. The generality can be realized by deducting the right-hand side from both sides, assuming that this does not reduce it.

To learn more about equations, refer:

https://brainly.com/question/1214333

#SPJ13

raise the difference of h and 4 to the 10th power

Answers

Answer:

h-4^10

Step-by-step explanation:

The difference of h and 4 is h-4

to the 10th power, is putting 10 as the exponent.

Marta went to the local discount store to stock up on school supplies. She bought four spiral notebooks at $1.50 each, a package of pens for $3.95, three felt tip markers at $.99 each, and a three-ring binder for $5.00. The sales tax is 6.5%. What was the total price Marta paid at the checkout counter?

Answers

Answer: 18.7598

Step-by-step explanation:

4 x 1.50= 6

6+ 3.95=9.95

3x 0.99=  2.97

2.97 + 9.95= 12.92

12.92+ 5= 17.92

6.5% of 12.92= 0.8398

6.5% of 12.92 + 17.92= 18.7598.

Five CD’s cost 80. If each CD cost the same, how much does one cost

Answers

Answer:

16

Step-by-step explanation:

Divide to find this answer.

If you're allowed to use a calculator, divide 80÷5 or

80/5 This will give you the answer 16.

If you are not permitted to use a calculator, see attached image for an example of a long division

find the f(x)=3x-2 domain on a graph

Answers

The domain of the  f(x)=3x-2 form a graph is -∞ < x<∞. See the attached graph. The interval notation is given as (-∞, ∞)

What is a domain?

It is to be noted that the domain of a function is the set of input or argument values for which the function is real and defined. Note from the graph that the function has no undefined points nor domain constraints. Hence, the domina is expressed as the notation:

-∞ < x < ∞.

If one were to solve for x,

Then

y = 3x -2

=

3x -2 = y [Flipping the eqation]
3x -2 + 2 = y + 2 {Added two to both sides]

Hence,

3x = y +2

Divide both sides by 3 to get:

3x/3 = (y+2)/3
x = [1/3y] + [2/3]

Learn more about the domain:
https://brainly.com/question/2264373
#SPJ1

(2x + 12y = −17) (3x + 2y = 7) value of x and y

Answers

Answer:

x = (59/16)

y = (-65/32)

Step-by-step explanation:

2x + 12y = −17

3x + 2y = 7

--------------------------

(2x + 12y = −17) × 3

6x + 36y = -51

(3x + 2y = 7) × -2

-6x - 4y = -14

----------------------------------

6x + 36y = -51

-6x - 4y = -14

-----------------------

32y = -65

÷32 ÷32

--------------------------

y = (-65/32)

3x + 2y = 7

3x + 2(-65/32) = 7

3x - (65/16) = 7

+(65/16) +(65/16)

-------------------------------

3x = (177/16)

÷3 ÷3

---------------------------

x = (59/16)

I hope this helps!

The quotient of a number y and 3, less 2 is between –7 and 4.

Answers

A statement is 2 less than the quotient of a number y and 3. The algebraic expression is (y/3)-2.

Given that,

A statement is 2 less than the quotient of a number y and 3.

According to question,

An algebraic expression can be written by use of 2  less than the quotient of a number y and 3.

To evaluate an algebraic expression is to determine its value when a certain number is used in lieu of the variable. In order to evaluate an expression, we first replace the variable with the supplied number, then we streamline the expression using the order of operations.

Here, expression is (y÷3)-2=(y/3)-2.

If we substitute the values 0,1,2,3,4...

We get the values between -7 and 4.

Therefore, the algebraic expression for the statement  2 less than the quotient of a number y and 3 is written as (y/3)-2.

To learn more about expression visit: https://brainly.com/question/14083225

#SPJ1

solve it if u can i bet u

Answers

The value of  x³(x³ - 18) if x(x - 3) = -1 is -1.

To simplify simply means to make anything easier. In mathematics, simplifying an equation, fraction, or problem means taking it and making it simpler. Calculations and problem-solving techniques simplify the issue.

We have been given that

x(x - 3) = -1 ....... eq(1)

Find out the value of

x³(x³ - 18) ........eq(2)

We first simplify the value of x in equation (1)

x(x−3)=−1

Divide by x on both side

x(x−3)/x = −1/x

(x−3)=−1 /x

Rewrite as

x + 1/x = 3

Taking cube on both sides

(x + 1/x)³ = 3³

x³ + (1/x)³ = 27 - 3×3

x³ + (1/x)³ = 27 -  9

x³ + (1/x)³ = 18

Multiply by x³

x^6 + 1 = 18x³

x^6 - 18x³ = -1

x³(x³ - 18) = -1

Hence, the value of equation x³(x³ - 18) is also -1.

Tp learn more about Simplification

https://brainly.in/question/4148085

#SPJ1

Please help TwT math is hard

Answers

Answer:

b

Step-by-step explanation:

The graph was horizontally stretched by a factor of 2.

You harvest 50 cherry tomatoes from your garden. you randomly inspect 15 tomatoes and find that 3 have bad spots on them. what is the experimental probability that a tomato has a bad spot?

Answers

Answer: 1 out of 5

Step-by-step explanation: 15 divided by 3 is 5. Which also means out of the 50 cherry tomatoes, 10 of them have bad spots.

Which of the following statements is true when comparing numbers using a number line?
The number closest to zero is always the least.
The number farthest from zero is always the greatest.
The number farthest right is always the least.
The number left is always the least.

Answers

From the given statements, the statement which is true when comparing numbers using a number line is :

The number left is always the least.

The definition of a number line is a straight line that shows numbers arranged at equal distances.

A number line is often shown horizontally and can be extended indefinitely in any direction.

Moving right causes the numbers to increase (or become greatest), while moving left causes the numbers to decrease ( or becomes least).

Larger numerals on the number line are always to the left of smaller numbers. Positive numbers have a negative opposite, and negative numbers have a positive opposite. Because it is defined as the distance from zero (the origin) on a number line, the absolute value of any real number is always positive.

Hence the right statement is that the number left is always the least.

Learn more about Number line here:

brainly.com/question/24644930

#SPJ1

D. The number left is always the least

QUESTION: William started with 40 pieces of candy and gave away x pieces. Hannah had 70 pieces of candy and gave away three times as many pieces as William did.

How many pieces of candy did William give away if they had the same number of pieces of candy left?


A. 30

B. 18

C. 15

D. 8

Answers

The pieces of candies that William gave away is 15.

How many pieces of candy did William give away?

The linear expression that can be used to determine the number of candies left is:

Pieces of candies left = initial pieces of candies - number of candies given out

Pieces of candies left when William gave out candies : 40 - x

Pieces of candies left when Hannah gave out candies : 70 - 3x

If they have the same pieces of candies left, the two above expressions would be equal.

40 - x = 70 - 3x

3x - x = 70 - 40

2x = 30

x = 15

To learn more about mathematical expressions, please check: https://brainly.com/question/11980704

#SPJ1

suppose you walk at the of 210 ft/min. you need to walm 10,000 ft. how many more minutes will it take you to finish if you have already walked 550 ft.

Answers

It will take you 45 more minutes to finish walking the remaining distance.

If you have already walked 550 ft, subtracting it to the total distance you need to walk, then the remaining distance will be:

10,000 ft - 550 ft = 9450 ft

Rate is the relationship between two quantities of different units. Rate of speed is the amount of distance over total time.

If you walk at the rate of 210 ft/min, then the time it will take you to finish the remaining distance will be:

rate = distance/time

time = distance/rate

time = 9450 ft/(210 ft/min)

time = 45 mins

Hence, it will take you 45 more minutes to finish the walk.

Learn more about rate of speed here; https://brainly.com/question/24873957

#SPJ4

The points P, Q, R, S, T, U, A and B on the number line are such that, TR = RS = SU and AP = PQ = QB. Name the rational numbers represented by P, Q, R and S.

Answers

The rational numbers are represented by P, Q, R, and S is mathematically given as

R= -4/3S= -5/3P= 7/3Q = 8/3

This is further explained below.

What are the rational numbers?

We make use of the idea of a number line, which displays the actual numbers in ascending order from left to right and from left to right to left.

One unit represents the distance between point U and point T.

It is separated into three sections that are all the same size.

Generally, the equation for TR is mathematically given as

TR = RS = SU

Therefore

TR = RS = SU = 1/3

[tex]R = -1 - (1/3)\\ R= -4/3\\\\S = -1 - (2/3) \\S= -5/3[/tex]

Considering the value of P

[tex]P = 2 + 1/3 = 6/3 + 1/3[/tex]

P= 7/3

Considering the value of Q

[tex]Q = 2 + 2/3 = 6/3 + 2/3[/tex]

Q = 8/3

Read more about rational numbers

https://brainly.com/question/24398433

#SPJ1

p²-q² = 7 and p-q=2. Find the value of p + q.​

Answers

7/2

Factor the quadratic and insert 2 for (p-q). Solve the new equation for (p+q).

See details:

Question 1 : given the replacement set {0,1, 2, 3, 4} . solve 5x - 6 = 9


Question 2: Given the replacement set {2, 3, 4, 5} . Solve 8x + 4 = 5x - 6

Answers

The solution to the equations are x = 3 and no solution in the set, respectively

How to solve the equations using the replacement sets?

Question 1

The equation is given as:

5x - 6 = 9

The replacement set is given as

{0,1, 2, 3, 4}

So, we replace the variables using the elements in the replacement set

When x = 0, we have

5(0) - 6 = 9

- 6 = 9 -- false

When x = 1, we have

5(1) - 6 = 9

- 1 = 9 -- false

When x = 2, we have

5(2) - 6 = 9

4 = 9 -- false

When x = 3, we have

5(3) - 6 = 9

9 = 9 -- true

Hence, the value of the equation is x = 3

Question 1

The equation is given as:

8x + 4 = 5x - 6

The replacement set is given as

{2, 3, 4, 5}

So, we replace the variables using the elements in the replacement set

When x = 2, we have

8(2) + 4 = 5(2) - 6

20 = 4 -- false

When x = 3, we have

8(3) + 4 = 5(3) - 6

28 = 9 -- false

When x = 4, we have

8(3) + 4 = 5(4) - 6

28 = 14 -- false

When x = 5, we have

8(5) + 4 = 5(5) - 6

44 = 19 -- false

Hence, the value of the equation is not in the solution set

Read more about equations at:

brainly.com/question/2972832

#SPJ1

Write the following number in standard decimal form. four and eight hundredths

Answers

Answer:

4.08.

Step-by-step explanation:

8/100

= 0.08

Determine whether y varies directly with x. If so, find the constant of variation and write the equation. x=-2,-4,-16 y= -4, -8, -32

Answers

For given data y varies directly with x and the constant of variation is 2.

And the equation is y = 2x

In this question, we have been given some values of x and y.

we need to determine whether y varies directly with x.

If so, find the constant of variation and write the equation.

x = -2, -4, -16

y = -4, -8, -32

Consider the ratio y/x

-4/-2 = 2

-8/-4 = 2

-32 / -16 = 2

As y/x ratio is constant, y varies directly with x.

and the constant of variation is 2.

Also, the equation from given x, y values would be,

y = 2x

Therefore, for given data y varies directly with x and the constant of variation is 2.

And the equation is y = 2x

Learn more about the equation here:

https://brainly.com/question/13170101

#SPJ1

PLEASE HELP I AM GROUNDED FOR MY GRADES

Answers

Answer:

a=41

Step-by-step explanation:

Answer:

a = 49

Step-by-step explanation:

41 + a + 90 = 180

solve for a

hope this helped

have a good day ^^

12y3 ― 4y3 = _____ help plez

Answers

Exponents and power - [tex]12y^3 - 4y^3 = 8y^3[/tex]

What are exponents and power?

Exponents and powers are two techniques for simplification of extremely big or extremely small numbers. For instance, we can write 3 x 3 x 3 x 3 as [tex]3^{4}[/tex], where 4 is the exponent and 3 is the base, to demonstrate 3 x 3 x 3 x 3 in a straightforward manner. There is a claim of power throughout the entire sentence [tex]3^{4}[/tex].

How do you perform exponentiation operations?

Rule 1: All operations enclosed in parenthesis must be simplified.
Rule 2 states that all exponents should be simplified, starting from the left.
Rule 3: When multiplying and dividing, always go from left to right.
Rule 4: Always add and subtract by moving from left to right.

[tex]8y^{3}[/tex] is the correct answer to the given equation.
[tex]12y^{3} - 4z^{3}\\ = (12-4)y^{3}\\= (8)y^{3}\\ =8y^{3}[/tex]

To learn more about exponents and powers visit:

https://brainly.com/question/11975096

#SPJ9

help PLEASE❗️❗️❗️❗️❗️❗️❗️

Answers

Answer:

lol its 2x your afh

Step-by-step explanation:

Other Questions
0.867, 0.086, 0.008, and 0.870 smallest to largest Demuestra los valores de las funciones trigonomtricas del ngulo de 0 Use the symbol < , = and > to compare each number.a) |-24| _ |-22|b) 18 _ |-28| The table represents a continuous exponential function f(x).x 2 3 4 5f(x) 12 24 48 96Graph f(x) and identify the y-intercept. 0 3 6 12 Help!A 2-column table with 5 rows. Column 1 is labeled Type of flower with entries lilies, daisies, carnations, asters, daffodils. Column 2 is labeled Amount with entries 60, 36, 72, 42, 24.The florist makes the greatest number of identical arrangements with the lilies and the daisies. Which combination describes the arrangements?A.6 arrangements with 10 lilies and 6 daisiesB.6 arrangements with 6 lilies and 10 daisiesC.12 arrangements with 10 lilies and 6 daisiesD.12 arrangements with 5 lilies and 3 daisies If (8^7)^2 = 2^y, what number is y? This table represents equivalent ratios. What are the values of a and b? An RNA virus is a virus that has RNA as its genetic material. The base compositions of two kinds of RNA viruses are the following:RNA virus (#1) 20.2% A, 20.0% U, 29.8% G, and 30.0% C.RNA virus (#2) 12.2% A, 37.8% U, 12.2.% G, and 37.8% C.Which of the following correctly describes the genetic materials of these two DNA viruses Can someone like explain to me how to get this. Samantha phoned herbrother. She explains thatshe needs to postpone hertrip to visit him, and sheasks if she can arrive laterin the month.O Samantha phoned her brother.She explained that she needed Which would best model the motion of P waves? I am a young women in the20's who was fashionable. Ibobbed my hair and woremy skirts where you could seemy ankles. Who am i ? About 2.4 billion years ago, the Great Oxidation Event occurred. During this time, the amount of oxygen in the atmosphere was estimated to go from trace concentrations to concentrations slightly higher than today. Explain what happened to photosynthetic organisms during the Great Oxidation Event.Provide examples of what may have occurred to the geosphere and hydrosphere during the Great Oxidation Event. what did gogol's wife mean when she states 'perhaps its just not enough to be bengali' 1) Draw a small graph on your paper, and graph the following line: y = 2x + 3 The midpoint of AB is M(-3, 5). If the coordinates of A are (-4, 6), what are thecoordinates of B? (6 ten thousands 3 hundreds) x 10 Jane wants to rent a boat and spend less than $75. The boat costs $8 per hour, and Jane has a discount coupon for $5 off. What are the possible numbers ofhours Jane could rent the boat?Use t for the number of hours.Write your answer as an inequality solved for t. A ball is launched with initial speed v from ground level up a frictionless slope. The slope makes an angle with the horizontal. Using conservation of energy, find the maximum vertical height hmax to which the ball will climb. An experience that would be listed in the Other Experience category would include _____.a.military serviceb.awards earnedc.leadership opportunitiesd.all of the above